La ricerca ha trovato 84 risultati

da gismondo
10 mar 2009, 18:44
Forum: Combinatoria
Argomento: Quale conviene?
Risposte: 8
Visite : 2796

Sisi il tuo ragionamento è giusto e il risultato è corretto. Io mi chiedevo dove fosse l'errore nell'altro ragionamento, quello che ho scritto sopra, che è sbagliato ma non ho ben chiaro in cosa...
da gismondo
10 mar 2009, 16:49
Forum: Combinatoria
Argomento: Quale conviene?
Risposte: 8
Visite : 2796

@ pakman ma se nella seconda metti 5/6 * 5/6 escludi il caso in cui ottieni un 1 e un numero diverso da 1...che invece va incluso perchè il quesito dice un doppio 1... @tutti gli altri mi chiedevo quale fosse l'errore in questo ragionamento: 1) il quesito dice almeno un 1, quindi impongo che esca un...
da gismondo
10 mar 2009, 15:03
Forum: Combinatoria
Argomento: Quale conviene?
Risposte: 8
Visite : 2796

Quale conviene?

Ci sono due giochini :
1) tiro 4 volte un dado, per vincere devo ottenere almeno un 1
2) tiro 24 volte 2 dadi, per vincere devo ottenere almeno un doppio 1
A quale mi conviene giocare?
da gismondo
08 mar 2009, 12:00
Forum: Combinatoria
Argomento: Giochi di archimede 2- numeri belli
Risposte: 11
Visite : 4853

Ma alla fine quanto viene? Anch'io non capisco bene perchè il rapporto di 1/3 si dovrebbe conservare anche dopo la limitazione.
da gismondo
06 mar 2009, 20:50
Forum: Teoria dei Numeri
Argomento: IMO 2007
Risposte: 11
Visite : 3259

(immagino tu intenda $ 4ab-1 $)
grazie per la risposta...però a me risulta $ 4ab-1 \ge 2a\pm1 $ per valori positivi di a e b, quindi come può essere $ ~4ab+1|2a\pm1 $ ?
da gismondo
06 mar 2009, 19:38
Forum: Teoria dei Numeri
Argomento: IMO 2007
Risposte: 11
Visite : 3259

Domanda un po' cretina... Ma se io scompongo (4a^2-1)^2 in questo modo: (2a-1)(2a-1)(2a+1)(2a+1) Questi sono tutte le forme possibili di divisori ? Cioè... si potrebbe far vedere che 4ab-1 non è nessuno di quegli oggetti lì? 4ab-1 \not= 2a \pm1 (a meno che a=b=1 ) 4ab-1 \not= (2a \pm1)^2 4ab-1 \not=...
da gismondo
02 mar 2009, 20:08
Forum: Teoria dei Numeri
Argomento: RMM 2009 - Problema 1
Risposte: 20
Visite : 6851

Fissiamo un primo ~\displaystyle p , e dimostriamo che la nostra frazione, ridotta ai minimi termini, non può avere un fattore ~\displaystyle p a denominatore. Così facendo, avremmo dimostrato che il denominatore è ~\displaystyle 1 , e quindi la frazione è in effetti un intero. Scusami, puoi chiari...
da gismondo
01 mar 2009, 19:59
Forum: Teoria dei Numeri
Argomento: Equazione diofantea
Risposte: 10
Visite : 3326

@SkZ
Grazie per il chiarimento.
da gismondo
01 mar 2009, 15:36
Forum: Teoria dei Numeri
Argomento: Equazione diofantea
Risposte: 10
Visite : 3326

Scusa, in che modo si utilizza l'algoritmo di euclide in questi casi? Io lo so utilizzare solamente per le equazioni in 2 variabili...
da gismondo
28 feb 2009, 14:46
Forum: Teoria dei Numeri
Argomento: quasi am-gm
Risposte: 5
Visite : 1880

AM è la media aritmetica mentre GM è la media geometrica (ovvero il prodotto degli n termini sotto radice n-esima) Esiste una disuguaglianza importante in merito : AM \ge GM col segno di uguaglianza che vale se e solo se tutti i termini sono uguali. In questo caso abbiamo 4 temini sotto una radice q...
da gismondo
27 feb 2009, 20:50
Forum: Teoria dei Numeri
Argomento: sns 1972-73
Risposte: 4
Visite : 3986

e vabbò va ... :D
$ (a+1)^2 = a^2 +2a +1 $ quindi il prodotto di 4 numeri consecutivi è della forma $ a(a+2) $ infatti:
$ n(n+1)(n+2)(n+3) = (n+1)(n+2)n(n+3)= (n^2 +3n +2)(n^2 +3n) $
da gismondo
22 feb 2009, 14:16
Forum: Teoria dei Numeri
Argomento: il 7 arriva prima o poi??
Risposte: 15
Visite : 5543

Ragazzi ma qualcuno che si cimenta nella dimostrazione? :P
In realtà il problema può essere allargato a "dimostrare che tutte le cifre compariranno prima o poi al primo posto"
da gismondo
22 feb 2009, 00:34
Forum: Matematica ricreativa
Argomento: 1=-1
Risposte: 7
Visite : 4690

Scusate ma ci sono alcune imprecisioni....:D
$ \sqrt a \cdot \sqrt b = \sqrt {ab} $
vale solo per $ a $ e $ b $ reali non negativi.
Inoltre la funzione radice quadrata principale restituisce solo valori $ \ge 0 $
da gismondo
21 feb 2009, 22:13
Forum: Teoria dei Numeri
Argomento: il 7 arriva prima o poi??
Risposte: 15
Visite : 5543

Se lo hai trovato con un ragionamento mi complimento, se lo hai trovato "brute-force" mi complimento ancora di più per la buona volontà :P
La dimostrazione? :D
da gismondo
21 feb 2009, 20:15
Forum: Teoria dei Numeri
Argomento: il 7 arriva prima o poi??
Risposte: 15
Visite : 5543

il 7 arriva prima o poi??

Problema molto interessante...
È stato proposto a una lezione tenuta a Roma3 riguardante le congruenze.

Data la funzione $ f(n)= 2^n $ dimostrare che per qualche valore di $ n $ il valore restituito ha come prima cifra (da sinistra) 7